Marcus' 2018 federal tax payment was $5,350. This payment was ⅕ of his 2018 annual salary. What is his 2018 annual salary?

Answers

Answer 1

Answer:

$26750

Step-by-step explanation:

1/5 * (total salary) = 5350

So, 5*5350 = total salary = 26750


Related Questions

Can someone help me?It's urgent and thank you!

Answers

Answer:

y = square root x2 - 5

Step-by-step explanation:

y=[tex]\sqrt{x} -5[/tex]

(the top option)

provide explanation too please !


a piano teacher teaches 8 lessons in 6 hours.
the lessons are all the same length.
how long is one lesson, in minutes?

Answers

1 hour is 60 minutes

6 hours x 60 = 360 total minutes

Divide total time by number of lessons:

360/8 = 45

Each lesson was 45 minutes

How do I do this?? TnT

Answers

Answer:

x = 4

Step-by-step explanation:

Corresponding angles are congruent

4x + 44 = 6x + 36

44 = 2x + 36

8 = 2x

x = 4

Put these numbers in order from least to greatest.
9.9, 9.5, and 9 4/5

Answers

9.5, 9 4/5, 9.9

9 4/5 is equal to 9.8

WILL GIVE BRAINLIEST
Hannah and Han are each trying to solve the equation x² – 8x + 26 = 0. They know that
x = -1 are i& - i, but they are not sure how to use this information to solve for x in their
equation.
Part 1- Here is Hannah's work:
x? - 8x + 26 = 0
X? – 8x = -26
Show Hannah how
to finish her work using completing the square and complex numbers.
Part 2- Han decides to solve the equation using the quadratic
formula. Here is the beginning of his
work
-b+V62-4ac
-(-8)+7-8)2–401|(26)
Finish using the quadratic formula. Simplify the final answer as much as possible.

Answers

Part one:

[tex]x^2-8x=-26[/tex]

Rewrite in the form [tex](x+a)^{2} =b[/tex]

[tex]\left(x-4\right)^2=-10[/tex]

[tex]\mathrm{For\:}f^2\left(x\right)=a\mathrm{\:the\:solutions\:are\:}f\left(x\right)=\sqrt{a},\:-\sqrt{a}[/tex]

Solve [tex]x-4=\sqrt{-10} : x=\sqrt{10} i+4[/tex]

Solve [tex]x-4=\sqrt{-10} : x=-\sqrt{10} i+4[/tex]

[tex]x=\sqrt{10}i+4,\:x=-\sqrt{10}i+4[/tex]

Part two:

[tex]x=\frac{-\left(-8\right)\pm \sqrt{\left(-8\right)^2-4\cdot \:1\cdot \:26}}{2\cdot \:1}[/tex]

Simplify [tex]\sqrt{\left(-8\right)^2-4\cdot \:1\cdot \:26}}: 2\sqrt{10} i[/tex]

[tex]=\frac{-\left(-8\right)\pm \:2\sqrt{10}i}{2\cdot \:1}[/tex]

Separate solutions

[tex]x_1=\frac{-\left(-8\right)+2\sqrt{10}i}{2\cdot \:1},\:x_2=\frac{-\left(-8\right)-2\sqrt{10}i}{2\cdot \:1}[/tex]

[tex]\frac{-(-8)+2\sqrt{10}i }{2*1} :4+\sqrt{10}i[/tex]

[tex]\frac{-(-8)+2\sqrt{10}i }{2*1} :4-\sqrt{10}i[/tex]

[tex]x=4+\sqrt{10}i,\:x=4-\sqrt{10}i[/tex]

The solutions are:-

[tex]x=4+\sqrt{10i}\\\\x=4-\sqrt{10i}[/tex]

What is the equation?

The definition of an equation is a mathematical statement that shows that two mathematical expressions are equal.

Here given equation is

[tex]x^2- 8x + 26 = 0\\\\x^2-8x=-26\\\\(x-4)^2=-10\\\\[/tex]

[tex](x-4)=[/tex]±[tex]\sqrt{-10}[/tex]

[tex]x=\sqrt{10}i+4\\\\x=-\sqrt{10}i+4[/tex]

So,

[tex]x=\frac{-b+-\sqrt{b^2-4ac}}{2a}\\\\x=\frac{8+-\sqrt{(-8)^2-4(1)(26)}}{2(1)}\\\\x=\frac{8+-\sqrt{(-8)^2-4(1)(26)}}{2(1)}\\\\x=\frac{8+-\sqrt{64-104}}{2}\\\\x=\frac{8+-2\sqrt{10i}}{2}\\\\x=\frac{2(4+-\sqrt{10i})}{2}\\\\x=4+\sqrt{10i}\\\\x=4-\sqrt{10i}[/tex]

Hence, the solutions are:-

[tex]x=4+\sqrt{10i}\\\\x=4-\sqrt{10i}[/tex]

To know more about the equation

https://brainly.com/question/12788590

#SPJ2

Triangle HIJ is similar to triangle KLM. Find the measure of side MK. Round your answer to the nearest tenth.

Answers

Answer:

[tex]MK\approx 87.7[/tex]

Step-by-step explanation:

By definition, similar polygons have corresponding sides in a constant proportion. Therefore, we can set up the following proportion (ratio of corresponding sides) to solve for [tex]MK[/tex]:

[tex]\frac{20}{13}=\frac{MK}{57},\\MK=\frac{57\cdot 20}{13}\approx \boxed{87.7}[/tex]

Answer:

87.7

Step-by-step explanation:

Like stated previously similar triangle have side lengths with common ratios

*Create a proportionality to solve for MK*

57/13 = MK / 20

Now solve for MK

Multiply each side by 20

57/13 * 20 = 87.7 ( rounded )

MK/20 * 20 = MK

We're left with MK = 87.7

the average length of string a and string d is 10x cm. the average length of string b and string c is 8x cm . the average length of strings b , c , d is 6x cm . find the length of string a in terms of x

Answers

Answer:

[tex]a = 18x[/tex]

Step-by-step explanation:

Given

[tex]\frac{1}{2}(a + d) = 10x[/tex]

[tex]\frac{1}{2}(b + c) = 8x[/tex]

[tex]\frac{1}{3}(b + c+d) = 6x[/tex]

Required

The value of (a)

We have:

[tex]\frac{1}{2}(a + d) = 10x[/tex] --- multiply by 2

[tex]\frac{1}{2}(b + c) = 8x[/tex] --- multiply by 2

[tex]\frac{1}{3}(b + c+d) = 6x[/tex] --- multiply by 3

So, we have:

[tex]\frac{1}{2}(a + d) = 10x[/tex]

[tex]a + d = 20x[/tex]

[tex]\frac{1}{2}(b + c) = 8x[/tex]

[tex]b + c = 16x[/tex]

[tex]\frac{1}{3}(b + c+d) = 6x[/tex]

[tex]b + c + d= 18x[/tex]

Substitute [tex]b + c = 16x[/tex] in [tex]b + c + d= 18x[/tex]

[tex]16x + d = 18x[/tex]

Solve for d

[tex]d = 18x - 16x[/tex]

[tex]d = 2x[/tex]

Substitute [tex]d = 2x[/tex] in [tex]a + d = 20x[/tex]

[tex]a + 2x = 20x[/tex]

Solve for (a)

[tex]a = 20x - 2x[/tex]

[tex]a = 18x[/tex]

Find two integers whose sum is 1 and product is -2
TWO INTERGES

Answers

Answer:

2 and -1

Step-by-step explanation:

sum = 2 + (-1)

= 1

product = 2 × ( -1)

= -2

Find the slope of the line that passes through (2 2) and (-1 -2)

Answers

Answer:

4/3

Step-by-step explanation:

The formula is (y1-y2) /(x1-x2)

Answer:

slope = [tex]\frac{4}{3}[/tex]

Step-by-step explanation:

Calculate the slope m using the slope formula

m = [tex]\frac{y_{2}-y_{1} }{x_{2}-x_{1} }[/tex]

with (x₁, y₁ ) = (2, 2) and (x₂, y₂ ) = (- 1, - 2)

m = [tex]\frac{-2-2}{-1-2}[/tex] = [tex]\frac{-4}{-3}[/tex] = [tex]\frac{4}{3}[/tex]

5. What is x in the diagram?

Answers

Answer:

B

Step-by-step explanation:

we rule out all other 3 by the simple fact that the side is 9

the answer is B hope this helps

Can someone tell me if this is the correct choice?

Answers

Answer:

Step-by-step explanation:

It's not the correct choice. If a root is given, that is the same thing as the solution. We go backwards from a solution to a factor, which is what you need to do here.

If x = 2i is the solution, then the factor is

(x - 2i). Likewise, with the other one.

If x = 3i is the solution, then the factor is

(x - 3i). The leading coefficient of 1 just sits outside the first set of parenthesis, and because multiplication is commutative, it doesn't matter which set you put first. Thus, the one you want looks like this:

f(x) = (x - 2i)(x - 3i) or

f(x) = (x - 3i)(x - 2i).

Brad bought a piece of industrial real estate for $192,345. The value of the real estate appreciated a constant rate per year. The table shows the value of the real estate after the first and second years:

Year:
1
2

Value (in dollars):
$200,038.80
$208,040.35

Which function best represents the value of the real estate after t years?
A. f(t) = 200,038.80(1.04)^t

B. f(t) = 200,038.80(0.04)^t

C. f(t) = 192,345(0.04)^t

D. f(t) = 192,345(1.04)^t

Answers

Answer:

D. f(t) = 192,345(1.04)^t

Step-by-step explanation:

I took the test and it was right.

Also that is the original price and when you look at exponential functions, the starting point or original price is always first the then rate of increase. The table just shows how it increased in year 1 and 2.

Hope this helps. :)

A function assigns the values. The function that best represents the value of the real estate after t years is f(t) = 192,345(1.04)^t. Thus, the correct option is D.

What is a Function?

A function assigns the value of each element of one set to the other specific element of another set.

The initial cost of industrial real estate is $192,345, while the cost after one year is $200,038.80. Therefore, the rate of appreciation is,

[tex]\$200,038.80 = \$192,345(1+R)^t\\\\\$200,038.80 = \$192,345(1+R)^1\\\\\dfrac{\$200,038.80}{\$192,345}=(1+R)^t\\\\1.04 = 1 + R\\\\R = 0.04[/tex]

Hence, the function that best represents the value of the real estate after t years is f(t) = 192,345(1.04)^t. Thus, the correct option is D.

Learn more about Function:

https://brainly.com/question/5245372

#SPJ2

Factorise a² - b²
.......

Answers

(a-b)(a+b)

Regards,
ArmyCee:)

[tex]\longrightarrow{\green{ (a + b)(a - b) }}[/tex] 

[tex]\sf \bf {\boxed {\mathbb {STEP-BY-STEP\:EXPLANATION:}}}[/tex]

[tex] ={a}^{2} - {b}^{2} [/tex]

[tex] = (a + b)(a - b)[/tex]

[tex]\bold{ \green{ \star{ \orange{Mystique35}}}}⋆[/tex]

Al lanzar un dado dos veces consecutivas. ¿Qué probabilidad hay de obtener primero un 3 y luego un numero par?

Answers

Answer:

The probability is 1/12.

Step-by-step explanation:

Number of elements in sample space is 6 . Even numbers are 2, 4 and 6 so the there are 3 three even numbers.

So, the probability of getting 3 on the first chance and then an even number in the second chance is

[tex]P = \frac{1}{6}\times \frac{3}{6}\\\\P = \frac{1}{12}[/tex]

Kayleigh has $4500 in a savings account at the bank that earns 0.8% interest per year. How much
interest will she earn in 3 years?

Answers

Answer:

Kayleigh will have a total of $4608.

Step-by-step explanation:

First, you use the formula, I=PRT (Interest=Principal, Rate, Time), then you distribute the numbers: (I=4500x0.8%x3) when you multiply them all, you get $108, then you lastly add 108 to 4500, and you get your final answer of $4608.

Kayleigh will earn $108 in interest over a period of 3 years.

Interest is the additional amount of money that is charged or earned on a principal amount of money. It is typically expressed as a percentage of the principal and is either charged when borrowing money or earned when investing or saving money.

To calculate the interest Kayleigh will earn in 3 years, we can use the formula for simple interest:

Interest = Principal x  Rate x Time

Given:

Principal = $4500

Rate = 0.8% = 0.008 (decimal form)

Time = 3 years

Plugging the values into the formula, we have:

Interest = $4500 x 0.008 x 3

Calculating the expression, we find:

Interest = $108

Therefore, Kayleigh will earn $108 in interest over a period of 3 years.

To know more about simple interests follow

https://brainly.com/question/29785550

#SPJ2

If R(x) = 2 – 3x – 1, find R(-2)

a. -3
b. 9
c. -9
d. -11

PLEASE HELP

Answers

Answer:

C

Step-by-step explanation:

C

The output of  [tex]R(x) = x^{2} - 3x- 1[/tex] when x = -2 is 9.

What is PEMDAS?

PEMDAS exists as an acronym for the terms parenthesis, exponents, multiplication, division, addition, and subtraction.

To estimate the value of R(-2)

Substitute the value of x = -2, then

[tex]R(-2) = (-2)^{2} - 3(-2) - 1[/tex]

By using the PEMDAS order of operations

Calculate exponents, [tex](-2)^2 = 4[/tex]

= 4 - 3(-2) - 1

Multiply and divide (left to right), 3(-2) = -6

= 4 - (-6) - 1

Add and subtract (left to right),

4 - (-6) - 1 = 9

Therefore, the value of R(-2) = 9.

To learn more about the value of a function refer to:

https://brainly.com/question/15712340

#SPJ2

Which numbers are divisible by 2?

A)826

B)270

C)271

Answers

Answer:

A

Step-by-step explanation:

Answer:

A and B, every even number is divisible by two, C is odd

the length of a pond is 1700 CM breadth is 14m and height is 1000 CM if a point is half filled calculate the volume of a water in the pond ​

Answers

Answer:

1190 m^3

Step-by-step explanation:

l = 1700 cm = 17 m

b = 14 m

h = 1000 cm = 10 m

Total volume = l × b × h

= 17 × 14× 10

= 2380 m^3

since it is half filled ,

Volume is half , so,

volume of water in pond = 2380 ÷ 2

= 1190 m^3

Jada asked some students at her school how many hours they spent watching television last week, to the nearest hour. Here are a box plot and a histogram for the data she collected.

If anyone trolls imma be very sad :(

Answers

Answer:

100

Step-by-step explanation:

Based on the histogram given :

The vertical axis gives the number of student in the school :

For the range :

0 - 5 = 40

5 - 10 = 30

10 - 15 = 20

15 - 20 = 5

20 - 25 = 3

25 - 30 = 2

Taking the sum :

(40 + 30 + 20 + 5 + 3 + 2) = 100

a box contains 6 dimes, 8 nickels, 12 pennies, and 3 quarters. what is the probability that a coin drawn at random is not a dime

Answers

Add all the coins to get total coins:

6 + 8 + 12 + 3 = 29 total coins

Subtract dimes to find total of the coins that are not dimes:

29 -6 = 23

Probability of not picking a dime is the number of coins that aren’t dimes over total coins:

23/29

write tge following numbers expanded forms.
23900068407​

Answers

Answer:

Step-by-step explanation:

20000000000+3000000000+900000000+60000+8000+400+7

Helppp and explain pleaseeeeee!!!!!!

Answers

7/3 is your answer!

A scale model of a sculpture has a scale of 2:53
The height of the model is 20cm.
Find the height of the actual sculpture.
Give your answer in m.

Answers

Answer:

ok so we can simpley this is 1:106

so we multiply

20*106=2120 this is cm so we make it meters...

21.2meters!!!

Hope This Helps!!!

Given the interval 0<θ<π/2. Find the angle θ which is formed by the line y = -2x+4 and y = 3x-3
Show your work as well, thank you!​

Answers

Answer:

[tex]\rm\displaystyle \theta = \frac{\pi}{4} [/tex]

Step-by-step explanation:

we want to find the acute angle θ (as θ is between (0,π/2)) formed by the line y=-2x+4 and y=3x-3 to do so we can consider the following formula:

[tex] \displaystyle\tan( \theta) = \bigg| \frac{ m_{2} - m_{1} }{1 + m_{1} m_{2} } \bigg | [/tex]

[tex] \rm \displaystyle \implies\theta = \arctan \left( \bigg | \frac{ m_{2} - m_{1} }{1 + m_{1} m_{2} } \bigg | \right)[/tex]

From the first equation we obtain that [tex]m_1[/tex] is -2 and from the second that [tex]m_2[/tex] is 3 therefore substitute:

[tex] \rm\displaystyle \theta = \arctan \left( \bigg | \frac{ 3 - ( - 2) }{1 + ( - 2) (3)} \bigg | \right)[/tex]

simplify multiplication:

[tex] \rm\displaystyle \theta = \arctan \left( \bigg | \frac{ 3 - ( - 2) }{1 + ( - 6)} \bigg | \right)[/tex]

simplify Parentheses:

[tex] \rm\displaystyle \theta = \arctan \left( \bigg | \frac{ 3 + 2 }{1 + ( - 6)} \bigg | \right)[/tex]

simplify addition:

[tex] \rm\displaystyle \theta = \arctan \left( \bigg | \frac{ 5 }{ - 5} \bigg | \right)[/tex]

simplify division:

[tex] \rm\displaystyle \theta = \arctan \left( | - 1| \right)[/tex]

calculate the absolute of -1:

[tex]\rm\displaystyle \theta = \arctan \left( 1\right)[/tex]

calculate the inverse function:

[tex]\rm\displaystyle \theta = \frac{\pi}{4} [/tex]

hence,

the angle θ which is formed by the line y = -2x+4 and y = 3x-3 is π/4

(for more info about the formula refer the attachment thank you!)

Consider two vector-valued functions,

[tex]\vec r(t) = \left\langle t, -2t+4\right\rangle \text{ and } \vec s(t) = \left\langle t, 3t-3\right\rangle[/tex]

Differentiate both to get the corresponding tangent/direction vectors:

[tex]\dfrac{\mathrm d\vec r(t)}{\mathrm dt} = \left\langle1,-2\right\rangle \text{ and } \dfrac{\mathrm d\vec s(t)}{\mathrm dt} = \left\langle1,3\right\rangle[/tex]

Recall the dot product identity: for two vectors [tex]\vec a[/tex] and [tex]\vec b[/tex], we have

[tex]\vec a \cdot \vec b = \|\vec a\| \|\vec b\| \cos(\theta)[/tex]

where [tex]\theta[/tex] is the angle between them.

We have

[tex]\langle1,-2\rangle \cdot \langle1,3\rangle = \|\langle1,-2\rangle\| \|\langle1,3\rangle\| \cos(\theta) \\\\ 1\times1 + (-2)\times3 = \sqrt{1^2 + (-2)^2} \times \sqrt{1^2+3^2} \cos(\theta) \\\\ \cos(\theta) = \dfrac{-5}{\sqrt5\times\sqrt{10}} = -\dfrac1{\sqrt2} \\\\ \implies \theta = \cos^{-1}\left(-\dfrac1{\sqrt2}\right) = \dfrac{3\pi}4[/tex]

Then the acute angle between the lines is π/4.

explain by step by step pls :( if u type something wrong ill report u

Answers

Answer:

∠ C = ∠BCD = 30°

Step-by-step explanation:

∠ EDF = ∠GFC  = 110°                [ corresponding angles ]

Now consider triangle BCF

∠FBC  + ∠ABC = 180°                [ straight line angle ]

∠FBC  + 100° = 180°

∠FBC  = 180 - 100 = 80°

∠BFC + ∠GFC = 180°                  [ straight line angle ]

∠BFC + 110° = 180°

∠BFC = 180 - 110 = 70°

Sum of angles of triangle is 180°

Therefore , in triangle BCF

That is ,

       ∠F + ∠B + ∠ C  = 180°

          70° + 80° + ∠C = 180°

           ∠C = 180 - 150 = 30°            

       

Josiah invests $360 into an account that accrues 3% interest annually. Assuming no deposits or withdrawals are made, which equation represents the amount of money in josiahs account, y, after x years?

a-y=360(1.3)^x
b-y=360(0.3^x
c-y=360(0.03)^x
d-y=360(1.03)^x

Answers

Answer:

y = 360 (1.03)^x

Step-by-step explanation:

Amount in account = principle * (1+ rate) ^ time

When the interest is annual

rate is in decimal form and time is in years

y = 360 ( 1+.03) ^x

y = 360 (1.03)^x

Complete the equation for the relationship between the weight and number of months

Answers

Y=5.5x because it just is

Help please guys if you don’t mind

Answers

Answer:

first

2/5m + 8/5

second

22/5

third

11

2 Points

(25x

0
The equation ya

represents Michelle's regular

130(x-38) +950 %> 38

hourly and overtime wage. Based on this equation, what is Michelle's

overtime hourly pay?

A. $30

B. $38

O C. $950

O D. $25

Answers

Answer:

The answer is "35.22".

Step-by-step explanation:

Let the given equation:

[tex]\to 130(x-38) +950 \% \times 38=0\\\\\to 130x- 4940+\frac{950}{100} \times 38=0\\\\\to 130x- 4940+9.5\times 38=0\\\\\to 130x- 4940+361=0\\\\\to 130x- 4579=0\\\\\to 130x= 4579\\\\\to x= \frac{4579}{130}\\\\\to x= 35.22[/tex]

Answer: $25

Step-by-step explanation: took the quiz

Tell whether the equation Y=Y true or false

Answers

Answer:

its true trust biggie on this one fam

Step-by-step explanation:

its true broski

Other Questions
How does creon feel about the state law? During the earthquake several buildings and infrastructure were totally destroyed. Many people were killed and there was a Tsunami watch for the island. The earthquake measured __________on the scale? What is a molecule containing only carbon and hydrogen called?O A. An inorganic moleculeO B. A hydrocarbonO c. A metallic moleculeD. An ionic molecule(answer is provided) :) Can anyone help me with this question Suppose that g(x)= f(x)+ 6. Which statement best compares the graph of g(x) with the graph of f(x)?A. The graph of g(x) is the graph of f(x) shifted 6 units down.B. The graph of g(x) is the graph of f(x) shifted to the right.C. The graph of g(x) is the graph of f(x) shifted 6 units to the left.D. The graph of g(x) is the graph of f(x) shifted 6 units up. For a particular first-order reaction, it takes 48 minutes for the concentration of the reactant to decrease to 25% of its initial value. What is the value for rate constant (in s -1) for the reaction The figure above is a regular hexas What is the value of x? HELP ANYONE?Now you will put what you learned into practice. Remember, some ways to improve word choice include:Use a thesaurus to find synonymsUse words that are specific, not vagueStay away from using verbs like: is, was, were, & areRewrite the paragraph below to include more descriptive and colorful language that suggests a mood of exuberance or excitement. Specifically, edit this paragraph for word choice. Feel free to use a thesaurus, but try to maintain your own personal voice. For example, dont use words you wouldnt normally use. Make sure to edit for errors in conventions before turning it in.Paragraph to rewrite:"I had the best day ever. First, I woke up. Then, I got a package in the mail. I opened it. It was the best gift ever. Then, my aunt came over with a birthday cake and presents for the party. All my friends arrived and we had a good time. Ill always remember this day." Find the measure of X i need help about this question of physics The absorption spectrum of sodium has a line at 589 nm. What is the energy of this line? (The speed of light in a vacuum is 3.00 x 10^8 m/s, and Planck's constant is 6.626 x 10^-34 Jos.)A. 8.89 x 10^26 JB. 5.09 x 10^14 JC. 3.37 x 10^-19 JD. 2.96 x 10^18 J To prepare for his quiz on CPR, Woo Jin listed the steps he learned in his online health class. 1. Check for consciousness. 2. Check for a pulse. 3. Call 911. 4. Perform 100120 chest compressions per minute. 5. Push hard and fast during compressions. 6. Perform CPR until the person starts breathing or help comes. Woo Jin got 5 out of 6 on his quiz. Which step did Woo Jin get wrong and why?step 2, because you do not check for a pulse if you are not trainedstep 3, because you call 911 only after five minutes of not breathingstep 5, because chest compressions should be hard and slowstep 6, because CPR is done only until the call to 911 is made pleeese help me in this i will put the higher points What is the role of RNA? n a combination redox reaction, two or more ____________ , at least one of which is a(n) ____________ , form a(n) ____________ . General Reaction: ____________ In a decomposition redox reaction, a(n) ____________ forms two or more ____________ , at least one of which is a(n) ____________ . General Reaction: ____________ In double-displacement (metathesis) reactions, such as precipitation and acid-base reactions, ____________ of two ____________ exchange places; these reactions ____________ redox processes.General Reaction: ____________ In solution, single-displacement reactions occur when a(n) ____________ of one ____________ displaces the ____________ of another. Since one of the ____________ is a(n) ____________ , a A circle has a radius of 6cm. What is the area of a sector subtended by an angle of 30 degrees What do the readings of voices of an emerging nation reveal about diversity within the new nation use at least three specific examples from the text your answer should be at least once complete paragraph Select the word that correctly completes the sentence.Yo Choose... ven, veo, voy,ver los tigres en el zoolgico. 3. If your answer in #2 is YES, cite pieces of evidence that show the credibility and reliability of the source.4. If your answer in # 2 is NO, support your answer. 5. What is the important information presented by the author? Give at least 2 pieces of information. Pa help po Please help me out on this oneplzzzzzzzzzzzzzzzzzzzzzzzzzzzzzzzzzzzzzzzzzzzzzzzzTry this triangle: 3ft, 4ft, and 1 ftIs the first plus second greater than the third? Is 3+4> 1? ________Is the second plus third greater than the first ? is 4+1 >3?___________Is the third plus first greater than the second? Is 3+1 >4_________Can it make a triangle? Explain why?